LSAT and Law School Admissions Forum

Get expert LSAT preparation and law school admissions advice from PowerScore Test Preparation.

 Zarie Blackburn
PowerScore Staff
  • PowerScore Staff
  • Posts: 48
  • Joined: Jan 18, 2018
|
#79581
Complete Question Explanation

The correct answer choice is (B).

Answer choice (A):

Answer choice (B): This is the correct answer choice.

Answer choice (C):

Answer choice (D):

Answer choice (E):


This explanation is still in progress. Please post any questions below!
 lathlee
  • Posts: 652
  • Joined: Apr 01, 2016
|
#43787
I am just totally lost on this one. Can anyone please help me. I don't even know how to eliminate the wrong one here . especially the Powerscore back paged answer doesn't really explain doesn't stick well with me this occasion (10-135)
 Adam Tyson
PowerScore Staff
  • PowerScore Staff
  • Posts: 5153
  • Joined: Apr 14, 2011
|
#43813
"Biologists have proposed two mechanisms that bacteria might use" - that's two hypotheses being discussed or suggested. The passage goes on to say that if one is correct, certain behavior would be observed, but if the other was correct, different behavior would be observed. Finally, the author says the evidence supports one over the other, presumably based on which behavior was actually observed. That's what answer B describes - two hypotheses, a way to tell which one is correct, and evidence supporting one over the other.
 lathlee
  • Posts: 652
  • Joined: Apr 01, 2016
|
#43815
Thank you so much for answering so late so i might get an important revelation for an important test
User avatar
 christinecwt
  • Posts: 74
  • Joined: May 09, 2022
|
#95670
Hi Team - may I know why Answer Choice C is incorrect given that Answer Choice B and C are so similiar - that the only difference in Answer Choice C is about the "flaw inherent" mentioned while Answer Choice B saying the other one is "more accurate"?

Many thanks!
 Adam Tyson
PowerScore Staff
  • PowerScore Staff
  • Posts: 5153
  • Joined: Apr 14, 2011
|
#97335
That's exactly why C is wrong, christinecwt - the third paragraph says nothing about any flaws in either hypothesis. Instead, it just explains what would be true in each case. In order to select answer C you would have to be able to point to the flaws mentioned in that paragraph. Can you find them? If not, cross that answer out!

Get the most out of your LSAT Prep Plus subscription.

Analyze and track your performance with our Testing and Analytics Package.